免費論壇 繁體 | 簡體
Sclub交友聊天~加入聊天室當版主
分享
返回列表 发帖
回复 1# aishuxue


在别的地方展开也行
$\frac{\sqrt{3}+1}{2}\approx\frac{1.732+1}{2}=1.366$,为了计算上的方便,可以取一个靠近的值,比如$\ln(3)\approx 1.099$

于是有
\[e^x=e^{\ln(3)}+e^{\ln(3)}(x-\ln(3))+...\]
\[e^x>3+3(x-\ln(3))\]
\[e^{\frac{\sqrt{3}+1}{2}}>3+3(\frac{\sqrt{3}+1}{2}-\ln(3))\]
\[\approx 3+3(1.366-1.099)=3.801>3.732\approx\sqrt{3}+2\]

TOP

返回列表 回复 发帖